====== Lecture 3 ====== {{ :math105-s22:notes:note_jan_25_2022_math105_3.pdf |note}} [[https://berkeley.zoom.us/rec/share/54Qr7VHBSiXzfMp6LH0TkV6BpqMBqrIja6onUg-DV6a4W4nt_4XK5FstD1u6xh4b.NZK87fNfKNqq9iaT | video]] Today we continue going over Tao's sequence of Lemma 7.4.2 - 7.4.11 ==== Lemma 7.4.2 ==== I will prove the easy case (1-dim), you will do the general case in HW. Let $E = (0,+\infty)$ be the open half space in $\R$. For any subset $A \In \R$, we need to prove that $$ m^*(A) = m^*(A \cap E^c) + m^*(A \cap E). $$ Let $A_+ = A \cap E$ and $A_- = A \cap E^c$, note that if $0 \in A$, then $0 \in A_-$. First, we note that $$ m^*(A) \leq m^*(A_-) + m^*(A_+)$$ by finite sub-additivity of outer-measure. (This is because any open cover of $A_-$ and open cover of $A_+$, union together form an open cover of $A$.) Next, we need to show that, for any $\epsilon>0$, we have $$ m^*(A) + 2\epsilon \geq m^*(A_-) + m^*(A_+).$$ The plan is the following. Given an open covering of $A$ by intervals $\{B_j\}_{j=1}^\infty$, such that $m^*(A) + \epsilon > \sum_j |B_j|$. We define $$ B_j^+ = B_j \cap (0,\infty) , \quad B_j^- = B_j \cap (-\infty, \epsilon / 2^j) $$ then $\{B_j^+\}$ forms a cover of open interval of $A_+$, similarly $\{B_j^+\}$ forms a cover of open interval of $A_-$. $|B_j^+| + |B_j^-| \leq |B_j| + \epsilon / 2^j$. Thus, we have $$ m^*(A) + 2\epsilon \geq \sum_j |B_j| + \epsilon \geq \sum_j |B_j^+| + \sum_j |B_j^-| \geq m^*(A_+) + m^*(A_-). $$ That finishes the proof for $n=1$ case. How to generalize to higher dimension? In the above discussion, we used the trick of $1 = \sum_j 1/2^j$, which is same trick as we prove outer-measure has countable sub-additivity. See the hint in Tao's Exercise for a two-step prove, that utilizes many results that we have proven. ==== Lemma 7.4.4 ==== We only prove some part here. * (c) If $E_1, E_2$ are measurable, then $E_1 \cup E_2$ and $E_1 \cap E_2$ are measurable. * (e) every open box, and every closed box, is measurable * (f) if a set has outer-measure zero, then it is measurable. Proof: * (c) Try the case first with $A = \{ x^2 + y^2 < 1 \} \In \R^2$ and $E_1 = \{x>0\}$ and $E_2 = \{y>0\}$, then we can partition $$ A = A_{++} \sqcup A_{+-} \sqcup A_{-+} \sqcup A_{--} $$ (cut the pie to 4 pieces), then we have finite additivity of outer-measure in this case. Such conclusion holds in general, assuming $E_1, E_2$ are measurable. * (e) express open box as intersections of translated half-spaces. * (f) Assume $E$ has outer-measure 0, then $0 \leq m^*(A \cap E) \leq m^*(E) = 0$, we can prove $m^*(A) \geq m^*(A \RM E) + m^*(A \cap E) = m^*(A \RM E)$ again by monotonicity. ==== Lemma 7.4.5 ==== finite additivity: if $\{E_j\}_{j=1}^N$ is a finite collection of **disjoint** measurable sets, then for any subset $A$, we have $$ m^*(A \cap (\cup_j E_j)) = \sum_j m^*(A\cap E_j) $$ Proof: again, we try the case of $N=2$ first, to get intuition. Let $B = A \cap (E_1 \cup E_2)$, and $B_1 = A \cap E_1 = B \cap E_1$, and $B_2 = A \cap E_2 = B\cap E_2$, so, we are trying to prove that $$ m^*(B) = m^*(B_1) + m^*(B_2) $$ since $E_1$ and $E_2$ are disjoint, we notice that $B_2 = B \RM E_1$, thus the above holds by measurability of $E_1$ applied to the test set $B$ ===== Discussion Time ===== Let's fill in the details of the above sketches.